Teoria liczb III -- powtórzenie PDF Drukuj Email
Zadania I
Wpisany przez Joachim Jelisiejew   
wtorek, 03 stycznia 2012 18:06

Zadania 
Zadania PDF.

Źródło zadań w texu.

 
%        File: podstawy-tl.tex
%     Created: Sun Dec 11 11:00 PM 2011 C
% Last Change: Sun Dec 11 11:00 PM 2011 C
\documentclass[10pt]{article}
\usepackage{amssymb}
\usepackage{amsmath}
\usepackage{amsthm}
\textwidth 16cm
\textheight 24cm
\oddsidemargin 0cm
\topmargin 0pt
\headheight 0pt
\headsep 0pt
\usepackage[polish]{babel}
\usepackage[utf8]{inputenc}
\usepackage[T1]{fontenc}
\usepackage{polski}
\usepackage{import}
%\usepackage{MnSymbol}
% ----------------------------------------------------------------
\vfuzz4pt % Don't report over-full v-boxes if over-edge is small
\hfuzz4pt % Don't report over-full h-boxes if over-edge is small
% THEOREMS -------------------------------------------------------
\newtheorem{thm}{Twierdzenie}[section]
\newtheorem{cor}[thm]{Wniosek}
\newtheorem{lem}[thm]{Lemat}
\newtheorem{defn}[thm]{Definicja}
\newtheorem{tozs}[thm]{Tożsamość}
\newtheorem{hyp}[thm]{Hipoteza}
\newtheorem{useless}[thm]{}
 
\newenvironment{sol}[1][Rozwiązanie. ]{
\vskip 3mm
\noindent\emph{#1}
 
}
{\hfill\par}
 
\newcounter{problem}
\newenvironment{problem}[1][Zadanie]{
\stepcounter{problem}
\vskip 3mm
\noindent{\textsc{\bfseries #1 \theproblem}}\\}
{\hfill\par}
 
\def\abs #1{\left\vert #1\right\vert}
 
\renewcommand{\angle}{\sphericalangle}
\renewcommand{\vec}[1]{\overrightarrow{#1}}
\renewcommand{\leq}{\leqslant}
\renewcommand{\geq}{\geqslant}
\renewcommand{\dots}{\ldots}
 
\subimport{./}{style-ml.sty}
\def\sectionwidth{10cm}
 
\def\headpicture{fireworks.png}
\def\author{czyli kolejny fascynujący tytuł.\\Joachim Jelisiejew}
\def\date{12 grudnia 2011}
\begin{document}
\setlength{\topmargin}{-2cm}
\section{Teoria liczb~--- powtórka}
 
\subsection{Powtórzenie teorii}
\paragraph{Reszty modulo $n$}
    Cel: chcemy stwierdzić, czy równanie ma rozwiązanie w~liczbach
    całkowitych. Moglibyśmy sprawdzić wszystkie, ale jest ich nieskończenie
    wiele\dots Natomiast reszt z~dzielenia przez $n$ jest skończenie wiele,
    więc możemy sprawdzić wszystkie.
 
    Na resztach da się sensownie działać: jeżeli $a \equiv b$ i~$c \equiv d$
    (wszystko $\mod n$) to
    \[
    a + c \equiv b + d \mod n\quad a - c \equiv b - d \mod n,\quad ac \equiv bd
    \mod n, \quad a^m \equiv b^m \mod n \hbox{ dla każdego }m\in \mathbb{Z}_+
    \]
    Z~dzieleniem trzeba ostrożnie: mamy $2\cdot 2 \equiv 4\cdot 2 \mod 4$, ale
    $2 \not\equiv 4 \mod 4$. Jednak:
    \begin{center}
        jeżeli $NWD(a, n)= 1$ i~$ak \equiv al \mod n$, to $k\equiv l\mod n$.
    \end{center}
 
Udowodniliśmy także
\begin{thm}[Małe twierdzenie Fermata]
    Jeżeli $p$ jest liczbą pierwszą, a~$a$ jest całkowite, to
    \[
    a^p \equiv a \mod p.
    \]
    Jeżeli $p\not\big| a$, to $a^{p-1} \equiv 1 \mod p$, innymi słowy $a\cdot
    a^{p-2} \equiv 1 \mod p$, więc $a^{p-2}$ możemy NIEFORMALNIE (na razie)
    traktować jako ``$a^{-1}\mod p$''. Kiedyś może pokażemy, że to ma sens,
    a~jeśli nie, to na studiach.
\end{thm}
\subsection{Zadania}
 
\begin{problem}
    Pokaż, że jeżeli $p$ jest pierwsza, to jedynymi rozwiązaniami równania
    $x^2 \equiv 1 \mod p$ są $1\mod p$ i~$-1\mod p$ (\emph{tzn. każda liczba
    całkowita $x$ spełniająca $x^2 \equiv 1$, przystaje do $1$
    lub $-1$ modulo $p$}).
 
    Podaj przykład, że bez założenia, że $p$ jest pierwsza, teza zadania nie byłaby
    prawdziwa.
\end{problem}
 
\begin{problem}
    Udowodnij, że jeśli $p$ jest pierwsza, a~$a$ całkowita niepodzielna przez
    $p$, to 
    \[a^{\frac{p-1}{2}} \equiv 1 \mod p\hbox{ lub }a^{\frac{p-1}{2}}\equiv
    -1 \mod p.\]
 
    Wywnioskuj, że sześciany liczb całkowitych dają z~dzielenia przez $7$
    reszty ze zbioru $\{0, 1, -1\}$. Co można powiedzieć o~$5$ potęgach, $6$
    potęgach itd.?
\end{problem}
 
\begin{problem}
    Udowodnij, że równanie $x^3 - y^3 = 2012$ nie ma rozwiązań w~liczbach
    całkowitych $x, y$.
 
    \emph{Wskazówka: skorzystać z~wyniku poprzedniego zadania.}
\end{problem}
 
\begin{problem}
    Udowodnij, że równanie $x^5 - y^5 = 2010$ nie ma rozwiązań w~liczbach
    całkowitych $x, y$.
 
    \emph{Wskazówka: skorzystać z~wyniku poprzedniego poprzedniego zadania.}
\end{problem}
 
\begin{problem}
    Niech $p>3$ będzie liczbą pierwszą, udowodnić, że
    \[
    p\big|6^{p-2} + 3^{p-2} + 2^{p-2} - 1.
    \]
    \emph{(bardzo nieformalnie znaczy to: $1/6 +1/2 + 1/3 - 1 = 0$.)
    trzeba skorzystać z~twierdzenia Fermata, więc trzeba wymnożyć przez
    coś\dots}
\end{problem}
 
\end{document}